Difference between revisions of "2010 AMC 10B Problems/Problem 5"

(Solution)
(Solution 2)
Line 11: Line 11:
 
Let's make a calendar to visualize the situation better.
 
Let's make a calendar to visualize the situation better.
  
\begin{table}[]
+
<math>\begin{table}[]
 
\begin{tabular}{lllllll}
 
\begin{tabular}{lllllll}
 
\hline
 
\hline
Line 20: Line 20:
 
29                      & 30                      & 31                      &                        &                        &                        &                         
 
29                      & 30                      & 31                      &                        &                        &                        &                         
 
\end{tabular}
 
\end{tabular}
\end{table}
+
\end{table}</math>
  
 
==See Also==
 
==See Also==
 
{{AMC10 box|year=2010|ab=B|num-b=4|num-a=6}}
 
{{AMC10 box|year=2010|ab=B|num-b=4|num-a=6}}
 
{{MAA Notice}}
 
{{MAA Notice}}

Revision as of 14:09, 27 December 2019

Problem

A month with $31$ days has the same number of Mondays and Wednesdays. How many of the seven days of the week could be the first day of this month?

$\textbf{(A)}\ 2 \qquad \textbf{(B)}\ 3 \qquad \textbf{(C)}\ 4 \qquad \textbf{(D)}\ 5 \qquad \textbf{(E)}\ 6$

Solution

In this month there are four weeks and three remaining days. Any 7 days must have exactly one Monday and one Wednesday, so it works if the last $31 - 4\cdot 7 = 3$ days have the same number of Mondays and Wednesdays. We have three choices: Monday, Tuesday, Wednesday; Thursday, Friday, Saturday; Friday, Saturday, Sunday. The number of days the month can start on are Monday, Thursday, and Friday, for a final answer of $\boxed{\textbf{(B)}\ 3}.$

Solution 2

Let's make a calendar to visualize the situation better.

$\begin{table}[] \begin{tabular}{lllllll} \hline \multicolumn{1}{|l|}{1} & \multicolumn{1}{l|}{2} & \multicolumn{1}{l|}{3} & \multicolumn{1}{l|}{4} & \multicolumn{1}{l|}{5} & \multicolumn{1}{l|}{6} & \multicolumn{1}{l|}{7} \\ \hline \multicolumn{1}{|l|}{8} & \multicolumn{1}{l|}{9} & \multicolumn{1}{l|}{10} & \multicolumn{1}{l|}{11} & \multicolumn{1}{l|}{12} & \multicolumn{1}{l|}{13} & \multicolumn{1}{l|}{14} \\ \hline \multicolumn{1}{|l|}{15} & \multicolumn{1}{l|}{16} & \multicolumn{1}{l|}{17} & \multicolumn{1}{l|}{18} & \multicolumn{1}{l|}{19} & \multicolumn{1}{l|}{20} & \multicolumn{1}{l|}{21} \\ \hline \multicolumn{1}{|l|}{22} & \multicolumn{1}{l|}{23} & \multicolumn{1}{l|}{24} & \multicolumn{1}{l|}{25} & \multicolumn{1}{l|}{26} & \multicolumn{1}{l|}{27} & \multicolumn{1}{l|}{28} \\ \hline 29 & 30 & 31 & & & & \end{tabular} \end{table}$ (Error compiling LaTeX. Unknown error_msg)

See Also

2010 AMC 10B (ProblemsAnswer KeyResources)
Preceded by
Problem 4
Followed by
Problem 6
1 2 3 4 5 6 7 8 9 10 11 12 13 14 15 16 17 18 19 20 21 22 23 24 25
All AMC 10 Problems and Solutions

The problems on this page are copyrighted by the Mathematical Association of America's American Mathematics Competitions. AMC logo.png